Nclex Review: Tough practice questions archer review 2

अब Quizwiz के साथ अपने होमवर्क और परीक्षाओं को एस करें!

The nurse is providing teaching to a student nurse about the immune system. Which of the following is the best example of natural adaptive immunity? A. Cell-mediated response B. Lymphocyte creation C. Inflammatory response D. The flu vaccine

Choice A is correct. Cell-mediated immunity is the best illustration of natural adaptive immunity. This immunity is spurred by cytokines and T-lymphocytes and doesn't include antibodies.

how to put on PPE

donning: hand hygiene, gown, mask, eye protection, gloves doffing: remove gloves, gown, wash hands, eye wear, mask, wash hands

The nurse is teaching a client about methotrexate (MTX). Which of the following statements should the nurse include? Select all that apply. "This medication may cause you to bruise more easily." "You will need to take folic acid with this medication." "You must remain upright for thirty minutes after taking a dose." "You should avoid receiving inactivated vaccinations." "Avoid large crowds and wash your hands frequently."

"This medication may cause you to bruise more easily." "You will need to take folic acid with this medication." "Avoid large crowds and wash your hands frequently." Methotrexate (MTX) is indicated for a variety of autoimmune conditions. This medication carries serious adverse effects such as pancytopenia (low red blood cells, white blood cells, and platelets). Thus, the client may bruise more easily and be at a higher risk of infection, so avoiding crowded areas and practicing good hand hygiene are essential. MTX antagonizes folic acid, and while a client is taking MTX, folic acid supplementation is typically prescribed. Additional Info ✓ Methotrexate is a folate antagonist used to treat conditions such as rheumatoid arthritis, psoriasis, and some cancers. ✓ Common drugs interactions that can occur with methotrexate include: ✓ Protein-bound drugs and weak organic acids such as salicylates, sulfonamides, phenytoin ✓ Penicillins, NSAIDs ✓ Live virus vaccines ✓ Theophylline ✓ Hepatotoxic drugs

The primary health care provider (PHCP) prescribes ondansetron 0.15 mg/kg IV to a pediatric client who weighs 40 lb. The medication label reads 2 mg/mL. How many milliliters will the nurse administer to the client? Fill in the blank. Round the answer to the nearest tenth.

1.36 mL = 1.4 mL

The following scenario applies to the next 1 items The nurse reviews a medical record of a client diagnosed with breast cancer Item 1 of 1 History Of Present Illness (HPI) Client is a 56-year-old female whose routine mammogram showed a 2.3 x 4.5 cm lobulated mass in her left breast. A diagnosis of infiltrating ductal carcinoma was made following a biopsy. hX:Body Mass Index: 33 Nulliparous Maternal history of breast cancer Does not drink and is a non-smoker Currently taking iron for iron-deficiency anemia Human Papillomavirus with genital warts For each portion of the clinical history, specify if it is a risk factor for breast cancer or not a risk factor Body Mass Index 33 Age of 56 Maternal history of breast cancer Iron-deficiency anemia HPV with genital warts Nulliparous

A higher body mass index (BMI) during peri- and postmenopausal states increases the risk for breast cancer. Obesity is defined as a BMI equal to or greater than 30. The client's BMI of 33 and being postmenopausal are significant to their risk of breast cancer. Further, advancing age, especially postmenopausal, is a risk factor for breast cancer. The age bracket of 50-59 is a risk factor (1 in 42 women). As the client's age advances, so do the risk. A first-degree family history of breast cancer is a high-yield risk factor because of its relevance to the BRCA ½ mutations. Iron-deficiency anemia is not a risk factor for breast cancer; neither is HPV. HPV is a risk factor for head-and-neck and cervical cancers. Nulliparity is a risk factor for breast cancer for some poorly understood reason, possibly because of cell differentiation. Additional Info Breast cancer is the most diagnosed cancer amongst women. A woman can significantly reduce her risk of cancer or have an optimal outcome by modifying risk factors (smoking cessation, maintaining a normal BMI, and eliminating alcohol consumption) and engaging in screening (mammography starting at age 40). Nursing care for breast cancer prevention includes advising for a clinical breast exam that should begin in the 20s and mammography occurring annually starting at age 40. Clinical breast exams are now preferred over breast self-exams because of the rates of false positives. Risk factors for breast cancer include - Female sex White and black women Postmenopausal Early menarche or late menopause Caucasian Obesity Family history Nulliparity Smoking and alcohol consumption Higher endogenous estrogen levels

The nurse is assessing a client with a chest tube for crepitus. Which assessment technique is most appropriate for the nurse to perform? A. Press down on the client's abdomen, releasing, and assessing for pain. B. Palpate the skin around the chest tube and observe for a crackling sensation. C. Auscultating the bowel sounds in each quadrant. D. Inspect the client's chest for an even rise and fall.

Choice B is correct. The nurse may assess for crepitus by palpating the skin around the chest tube and observing for a crackling sensation. Crepitus is defined as infiltration of air in the subcutaneous layer of the skin, also known as subcutaneous emphysema. It is caused by air leaking into the subcutaneous space.

The ABCDEs of melanoma identification include which of the following? Asymmetry: one half does not match the other half Birthmark: cafe au lait spot that does not fade Color: pigmentation is not uniform Diameter: greater than 6 mm Evolving: any change in size, shape, color, elevation, or any new symptom such as bleeding, itching, or crusting

Choices A, C, D, and E are correct. ABCDE stands for: asymmetry, border, color, diameter, and evolution.

The nurse is caring for a client with angle-closure glaucoma. Which prescription should the nurse anticipate from the primary healthcare provider (PHCP)? A. Acetazolamide B. Diphenhydramine C. Phenylephrine D. Nortriptyline

Choice A is correct. Acetazolamide is a diuretic given intravenously or orally to a client with angle-closure glaucoma. Angle-closure glaucoma is a medical emergency that may cause blindness if not promptly treated. Acetazolamide causes a reduction in aqueous humor production by directly inhibiting carbonic anhydrase. As a result, it helps treat angle-closure glaucoma with intraocular pressure (IOP) greater than 30 mmHg. Acetazolamide acts within one hour of administration, and its therapeutic effects peak within four hours. Choices B, C, and D are incorrect. Anticholinergics, tricyclic antidepressants (TCAs), decongestants, and antihistamines should be avoided because these medications raise the IOP. Glaucoma is the leading cause of blindness in North America, and the client's medications must be reviewed so the client avoids taking these types of drugs. Diphenhydramine is an antihistamine with anticholinergic effects and should be avoided in glaucoma (choice B). Systemic phenylephrine is a sympathomimetic that is used as a nasal decongestant. Topical phenylephrine is also used in decongestant eye drops. This drug stimulates alpha-1 and alpha-2 adrenergic receptors and can cause transient mydriasis (pupillary dilation). Pupillary dilation exacerbates angle closure and increases IOP. In clients with narrow angles, such mydriasis can trigger an attack of acute angle-closure glaucoma (choice C). Nortriptyline is a TCA. TCAs have anticholinergic side effects and are avoided in angle closure glaucoma (choice D).

You are working in a community clinic. You are giving instructions to a 72-year-old man who was diagnosed today with early bilateral senile cataracts. You know that the man understood your instructions when he says: A. "I may have to quit driving until I get the cataracts treated." B. "I am going to miss being able to read the morning newspaper." C. "My wife will have to pick out my clothes since I won't be able to see the colors." D. "I will have to be careful since my eyes won't move together."

Choice A is correct. As individuals age, they are at increased risk for senile cataracts. During the early stages of this condition, diminishing distance vision is the highest risk for older adults. The nurse must caution the patient that the ability to see signs when driving will present a significant risk. Choice B is incorrect. Usually, near vision is not affected by these cataracts. Choice C is incorrect. In the early stages, color vision is not usually affected, although there may be changes as the condition progresses. Choice D is incorrect. Extraocular movements are not affected by senile cataracts. The cranial nerves that control eye movements are not affected by cataracts.

The nurse is caring for a client who has a vaso-occlusive event (VOE) secondary to sickle cell disease (SCD). Which of the following would indicate the client is achieving the treatment goals? A. Decreased self-report of pain. B. Increased white blood cell (WBC) count. C. Decreased energy while ambulating. D. Increased skin tenting in the sternum.

Choice A is correct. Classic manifestations associated with a vaso-occlusive crisis include diffuse pain and signs of dehydration. Choices B, C, and D are incorrect. Increased WBCs is a classic finding for a client experiencing a VOE. This is caused by the inflammation occurring with this disease process. Fatigue is a common complaint associated with the disease process overall. This is not specific to a VOE which is manifested by hypovolemia and pain. Increased skin tenting in the sternum would be evidence of dehydration and would indicate the client is not meeting the treatment goals, which involves repleting fluids.

Which of the following clinical manifestations of the aging immune system should alert the nurse to increased susceptibility to illness in elderly clients? A. Increased autoimmune responses B. Increased production of T and B cells C. Increased lymphoid tissue D. Increased circulation of lymphocytes

Choice A is correct. Elderly clients experience an increased autoimmune response that puts them at higher risk for experiencing diseases such as rheumatoid arthritis and other collagen-related diseases. Choice B is incorrect. The number of T and B cells produced decreases with age, which makes the immune system less efficient. Choice C is incorrect. Lymph tissue is also decreased, resulting in lower immune responses. Choice D is incorrect. The number of circulating lymphocytes in the elderly is reduced by 15%, along with a decline in antibody-antigen reaction, making the elderly more susceptible to infection.

The nurse is caring for a client who is prescribed enoxaparin. Which laboratory value should the nurse monitor? A. Platelet count B. Activated Partial Thromboplastin Time (aPTT) C. International Normalized Ratio (INR) D. Troponin

Choice A is correct. Enoxaparin is a low molecular weight-based heparin (LMWH). One of the adverse events of enoxaparin is heparin-induced thrombocytopenia (HIT). This severe condition results in a 50% or more decrease in the platelet count while also causing thrombosis. Therefore, it is reasonable to monitor the platelet count after initiating enoxaparin. Choices B, C, and D are incorrect. Unlike unfractionated heparin, enoxaparin does not require aPTT monitoring (choice B). aPTT is appropriate to monitor a client receiving unfractionated heparin. An unfractionated heparin dose is titrated for an aPTT of 1.5 to 2.5 times the control (baseline) value to achieve the treatment effect. The INR (choice C) is a monitoring parameter for warfarin. Finally, troponin is a lab test for individuals with suspected myocardial infarction (choice D). The nurse should be concerned if the troponin was elevated, indicative of injury to the myocardium. None of these labs require monitoring during enoxaparin therapy.

The nurse is interviewing a client who is assessed to have poor muscle coordination, stooped posture, and slow movements. Which medication on the client's daily medication list would most likely cause these findings? A. Haloperidol B. Nifedipine C. Venlafaxine D. Prazosin

Choice A is correct. Haloperidol is a typical antipsychotic which may adversely cause extrapyramidal side effects (EPS). These effects include akathisia, dystonia, pseudo parkinsonism, and/or tardive dyskinesia. Choices B, C, and D are incorrect. Nifedipine is a calcium channel blocker indicated for the treatment of hypertension. It also may be used as a tocolytic to mitigate preterm labor. Venlafaxine is a serotonergic drug used in the management of depressive and anxiety disorders. Prazosin is indicated for the treatment of hypertension as well as PTSD.

The mother of a 2-year-old boy states to the nurse during their check-up: "I just don't get it. He just sits there and plays on his own while all his other cousins play with each other. Is there anything wrong with him?" Which response by the nurse is most appropriate? A. "Your child is a toddler. It's normal for his age to just play all by himself while other children play too." B. "Did you encourage him to play with the other children? Maybe you don't encourage him that's why he doesn't play with them." C. "Let's mention that to the doctor when he comes in to see him." D. "I really recommend your child be checked by a child psychologist."

Choice A is correct. It is usual for toddlers to play by themselves and not interact with each other. This is called "parallel play." Choice B is incorrect. This statement is blaming the mother and makes her feel guilty for her child's behavior. The nurse should not mention this statement. Choice C is incorrect. This is normal behavior for the toddler. There is no need for the nurse to mention this situation to the physician. Choice D is incorrect. There is no need to refer the child to a child psychologist regarding the child's behavior. Although a session with the child psychologist would be helpful for the mother to understand her child's behavior better; however, it is not necessary.

A client presents to the clinic asking the nurse about emergency contraception. The nurse anticipates that the primary healthcare provider (PHCP) will prescribe which medication? A. Levonorgestrel B. Tamoxifen C. Finasteride D. Methotrexate

Choice A is correct. Levonorgestrel (LNG) is available over the counter for emergency contraception. This medication is indicated to be used up to 72 hours following unprotected intercourse, where pregnancy could be possible. It may be used off-label up to 120 hours following the event. This medication works by postponing (or inhibiting) ovulation. Choices B, C, and D are incorrect. These medications are not indicated as emergency contraception. ➢ Tamoxifen is indicated for hormone receptor-positive breast cancer. This medication is used to prevent breast cancer recurrence. ➢ Finasteride is indicated for benign prostatic hyperplasia. This medication is highly teratogenic and should be handled with gloves by pregnant women. ➢ Methotrexate is indicated for ectopic pregnancy that has not ruptured. This may be an alternative to surgery.

A client diagnosed with acute gastroenteritis is prescribed a 40 mEq potassium chloride capsule for hypokalemia. Which action should the nurse take when administering this medication? A. Open the capsule and sprinkle on a spoonful of applesauce B. Instruct the client to chew the capsule C. Give separate from other medications D. Give the medication two hours before meals

Choice A is correct. Potassium supplements can be distasteful, even in capsule form. To improve palatability, the nurse may sprinkle the contents of the potassium capsule on apple sauce, and the client can then swallow it. This also should be used for those who have difficulty swallowing. The client must not chew on a capsule or tablet. Choices B, C, and D are incorrect. The potassium capsule should not be chewed or added to hot foods (they will dissolve). Potassium does not have to be administered separately from other medications. Finally, potassium should be administered with food (and plenty of water) to prevent gastrointestinal irritation. Potassium chloride may be given orally or intravenously ➢ Intravenous potassium should never be administered by IV push ➢ Intravenous potassium is given at a maximum of 10 mEq/hr for a peripheral IV; 20 mEq/hr for a central line ➢ An infusion pump should always be used ➢ The medication should be administered into a patent intravenous (IV) catheter; patency needs to be assessed before IV potassium is administered ➢ If potassium chloride is given by elixir, it should be diluted in a minimum of 120 mL of cold water ➢ The potassium chloride tablet may be broken in half, and each half swallowed separately. May also dissolve the whole tablet in approximately 4 ounces of water ➢ The tablet should not be crushed ➢ Capsules may be opened up on pudding or applesauce. They should not be chewed. ➢ Capsules should be taken with a full glass of water and a snack to avoid GI irritation.

What action does the nurse perform to follow safe technique when using a portable oxygen cylinder? A. Check the amount of oxygen in the cylinder before using it. B. Use a cylinder for a patient transfer that indicates available oxygen is at 500 psi. C. Place the oxygen cylinder on the stretcher next to the patient. D. Discontinue oxygen flow by turning the cylinder key counter-clockwise until it is tight. Submit Answer

Choice A is correct. The cylinder must always be checked before use to ensure that enough oxygen is available for the patient.

The patient with history of right mastectomy is receiving maintenance IV fluids via peripherally inserted intravenous line in the left cephalic vein. The patient complains of pain at the IV site, and the nurse notes that the infusion has slowed and assesses swelling and erythema at the IV site. Which action should the nurse take first? A. Stop the infusion and remove the IV catheter B. Insert new IV in left intermediate basilic vein. C. Prepare the patient for PICC line placement. D. Elevate the right arm to reduce swelling.

Choice A is correct. This patient's IV site shows signs of phlebitis: redness, swelling, pain, and slowed infusion rate. The first priority action is to remove the current IV catheter to reduce the risk of further complications. Localized symptoms of phlebitis typically resolve after discontinuation of the catheter. Choice B is incorrect. Since this patient is not a candidate for IV access in the opposite arm due to a history of right mastectomy, the nurse should remove the current IV, and then attempt to insert a new IV proximal from the original site, but the current IV site should be discontinued first, prior to initiating any other interventions. Choice C is incorrect. This patient may be a candidate for PICC line placement if attempts to insert IVs at new sites are unsuccessful, but the current IV site should be discontinued first, prior to initiating any other interventions. Choice D is incorrect. Although phlebitis symptoms can be relieved by elevating the affected limb, applying a warm compress application, and administering analgesics, the current IV site should be discontinued first, prior to initiating any other interventions.

The patient recovering from hip surgery needs to regain strength in order to climb the flight of stairs leading to their bedroom at home. The nurse would expect which facility staff member to treat this patient's physical disability? A. Physical therapist B. Nutritionist C. Case Manager D. Occupational therapist

Choice A is correct. This patient, who has recently had major surgery that will affect their strength and ability to move, will most benefit from the services of a physical therapist. Physical therapists work to test, examine, and treat disabilities.

The nurse is observing a nursing student prepare to give an intramuscular (IM) injection in a client's deltoid. Which action by the student requires follow-up? A. Depresses the plunger of the syringe during the insertion of the needle B. Locates the injection site 3 finger widths below the acromion process C. Positions the client either lying or sitting position D. Dons clean gloves before administering the injection

Choice A is correct. This technique is not appropriate and requires follow-up. Depressing the plunger during the needle insertion is inappropriate because the medication is discharged when not in the appropriate spot. Once the nurse has appropriately anchored the needle in the appropriate landmark, the nurse can then depress the plunger, ensuring that the medication is discharged in the appropriate location. Choices B, C, and D are incorrect. These actions are correct and do not require follow-up. When giving an IM in the deltoid, the injection site is 3 finger widths below the acromion process. The client should sit or lie down when giving an IM in the deltoid. This best practice allows the nurse to appropriately locate the anatomical landmarks. Additionally, if the client should develop syncope from the injection, this also prevents injury. Clean gloves should be worn for this procedure as the likelihood of coming into contact with blood is high.

The RN performs palpation and percussion in a head-to-toe assessment. Over what organ would he/she expect to hear tympany when percussed? A. Stomach B. Liver C. Normal lung tissue D. Tympany is an abnormal finding

Choice A is correct. Tympany refers to a high, loud, drum-like tone that can be heard with percussion over air-containing organs. The stomach and intestines would produce tympany in a healthy adult.

You are caring for an 8-month-old infant with a tracheostomy. Upon assessment, you visualize secretions within the tracheostomy that require suctioning. In preparation to suction the infant's tracheostomy, which of the following settings would be the most appropriate suction setting? A. 120 mmHg B. 90 mmHg C. 60 mmHg D. 40 mmHgChoice

Choice B is correct. 90 mmHg would be the most appropriate suction setting for an 8-month-old infant based on the choices provided. For infants and children up to 24 months, tracheostomies should be suctioned using 80-100 mmHg. Choice A is incorrect. A suction setting of 120 mmHg is the upper limit for children aged 24 months and older. Therefore, this suction setting would be too powerful to utilize on an 8-month-old infant. Choice C is incorrect. 60-80 mmHg is the recommendation for suctioning newborns and neonates (i.e., up to 28 days). Therefore, 60 mmHg will likely not provide adequate suctioning power for this client. Choice D is incorrect. Attempting to suction a tracheostomy using 40 mmHg would not produce enough suctioning power to render a viable result. This pressure is not indicated for any age range.

The nurse is caring for a child who is immunocompromised and diagnosed with varicella. The nurse should expect a prescription for which medication? A. Amoxicillin-Clavulanate B. Acyclovir C. Doxycycline D. Azithromycin

Choice B is correct. Antiviral medications such as acyclovir or valacyclovir are commonly used to treat varicella infections. While these medications are not routinely prescribed for all infections, immunocompromised individuals are at risk for varicella complications, including meningitis. Thus, antiviral medications would be appropriate in this circumstance. Choices A, C, and D are incorrect. These medications are antibiotics and are not clinically indicated in the treatment of varicella.

The nurse is reviewing the assessment data for a client with acute glomerulonephritis (AGN). Which of the following would be an expected finding? A. Ketonuria B. Hematuria C. Polyuria D. Glycosuria

Choice B is correct. Clinical features of acute glomerulonephritis (AGN) include proteinuria, hematuria, periorbital edema, weight gain, high blood pressure, and decreased glomerular filtration rate (GFR). Choices A, C, and D are incorrect. Individuals with glomerulonephritis would have oliguria and not polyuria. This is explained because of the massive inflammation occurring in the glomerulus. Glycosuria and ketonuria are not features of this disease; instead, these may be expected in a client with uncontrolled blood glucose.

he emergency department (ED) nurse cares for a client who presents with irritability, nuchal rigidity, and a fever. Which of the following actions should the nurse take first? A. Administer prescribed ibuprofen. B. Place the client on droplet precautions. C. Notify the public health department. D. Obtain prescribed blood cultures.

Choice B is correct. Initiating droplet precautions is a high priority for this client. The classic bacterial meningitis triad is fever, neck stiffness, and altered mental status. The nurse must protect the other clients and staff from disease transmission. Thus, the nurse should initiate droplet precautions by placing the client in a room with all visitors and staff wearing surgical mask in the client's presence.

The client with chronic obstructive pulmonary disease (COPD) reports trouble sleeping at night. Which question is most important for the nurse to ask? A. "What do you eat before you go to bed? B. "How many pillows do you sleep on at night?" C. "Have you always been a light sleeper?" D. "Is your partner snoring and keeping you awake?"

Choice B is correct. Orthopnea is shortness of breath that occurs when lying flat, causing the person to have to sleep propped up in bed or sitting in a chair. Asking the client how many pillows they use to sleep on is a way to assess if the client has been educated about measures to prevent orthopnea. COPD causes blocked or narrowed airways that make breathing more difficult. Clients may experience symptoms like wheezing, coughing, mucus production, and tightness in the chest. Smoking or exposure to harmful chemicals can cause COPD. Orthopnea is a common symptom of COPD clients.

The nurse is completing an assessment on a 6-year-old client with asthma. Which of the following assessment findings is of most concern to the nurse? A. Expiratory wheezing B. Silent chest C. Cough D. Head bobbing

Choice B is correct. Silent chest is the assessment finding of most concern. This refers to the inability to auscultate any lung sounds. There is complete obstruction of the patient's airway and therefore the inability to move air. When complete obstruction occurs, this is a medical emergency. This assessment finding is of most concern because the client has lost their airway.

The nurse is assessing a 2-year-old client with the following symptoms: excessive drooling, stridor, difficulty swallowing, and difficulty speaking. Based on these assessment findings, which condition does the nurse suspect? A. Croup B. Epiglottitis C. Laryngotracheal bronchitis D. Bronchiolitis

Choice B is correct. The cardinal signs of epiglottitis are the "4 Ds" - drooling, dysphonia, dysphagia, and distress. Difficulty swallowing is dysphagia and difficulty speaking is dysphonia. Stridor is a high-pitched wheezing sound caused by disrupted airflow, hence the distress. This child is presenting with all of those cardinal symptoms and is therefore highly suspicious of epiglottitis.

A client complains of persistent fever and urinary urgency, prompting the health care provider (HCP) to order the client to provide a urine specimen for urinalysis, culture, and sensitivity analysis. The nurse ensures that the client collects the specimen properly by informing the client to collect the specimen from: A. The first stream of urine from the bladder B. The midstream urine from the bladder C. The final stream of urine from the bladder D. The entire volume of urine from the bladder

Choice B is correct. The preferred method of urine specimen collection for this client is known as a clean catch midstream urine sample. When clients are ambulatory and competent, this is most often a self-obtained specimen in a private bathroom. If the urine is not collected in a sterile or clean catch manner, the urine sample may be contaminated by bacteria originating from the skin or genital region and not from the urinary tract. This is often described by the clinical laboratory as mixed growth bacteria. A contaminated sample may lead to a false-positive urine culture result. The likelihood of mixed growth bacteria contamination is decreased by instructing the client to collect the specimen from the midstream portion of the client's void.

During a holiday party in your long-term care facility, you take unplanned and surprise candid photographs of residents enjoying the festivities for the monthly newsletter. What have you done? A. You have appropriately facilitated reminiscence therapy with photographs. B. You have violated federal law. C. You have effectively facilitated group cohesiveness with these photographs. D. You have violated state law.

Choice B is correct. You have violated the federal law entitled the Health Insurance Portability and Accountability Act (HIPAA) of the federal government. The Health Insurance Portability and Accountability Act mandates confidentiality and protects the clients' right to confidentiality. Taking photographs without resident consent is a violation of the federal Health Insurance Portability and Accountability Act. Choice A is incorrect. You have not appropriately facilitated reminiscence therapy with these photographs. These surprise group photographs are NOT appropriate. Choice C is incorrect. You have not appropriately facilitated group cohesiveness with these photographs. These surprise group photographs are NOT appropriate. Choice D is incorrect. You have not violated state law, but you have violated federal law. Health Insurance Portability and Accountability Act is governed by federal law.

The nurse educates a client about the application of a plaster cast to a fractured radius. Which of the following statements by the client would require follow-up? A. "If my arm feels itchy, I can use a hair dryer on the cool setting for relief." B. "I can reduce my arm's swelling by elevating it with a pillow." C. "I should be okay to shower with my cast." D. "It will be normal for me to feel heat after the cast is applied."

Choice C is correct. A plaster cast should be kept clean and dry. The client stated that he could shower with the cast, which requires follow-up. The only way a client should shower with this particular cast would be with a waterproof bag affixed over the cast. If that is not available, and the client has a fractured arm, they should take a bath and keep the affected extremity outside the tub. Choices A, B, and D are incorrect. These statements are correct and do not require follow-up. A casted extremity may cause skin irritation which causes a client to itch. The client should not stick any objects under the cast; they should try blowing cool air from a hair dryer instead. Elevating the extremity on a pillow is encouraged to help reduce swelling. Plaster casts are not as commonly utilized as fiberglass because a key disadvantage is that a plaster cast may take up to 48 hours to dry completely. As a cast is drying, it is normal for a client to feel a warm/heat sensation.

The nurse is caring for a client diagnosed with attention deficit hyperactivity disorder (ADHD). The nurse should anticipate a prescription for which medication? A. Citalopram B. Risperidone C. Methylphenidate D. Carbamazepine

Choice C is correct. ADHD may be treated by psychostimulants such as amphetamines or methylphenidate. These medications work by projecting dopamine and norepinephrine in the front of the brain to ameliorate the symptoms of inattention, impulsivity, and hyperactivity. Choices A, B, and D are incorrect. Citalopram is a serotonergic drug used in the treatment of depressive and anxiety disorders. Risperidone is indicated for psychotic disorders such as schizophrenia. Carbamazepine is an anticonvulsant indicated for bipolar disorders as it has a mood-stabilizing effect.

You are the nurse in a pediatrician's office. An 8-year-old boy with a history of asthma is brought to the office with complaints of a drippy nose, congestion, and runny eyes. The NP sees the patient and makes the diagnosis of allergic rhinitis. The NP prescribes an intranasal corticosteroid and an intranasal antihistamine. The outcome from allergic rhinitis that would put this child at highest risk is: A. Impaired sleep B. Decreased school performance C. An asthma attack D. Irritability

Choice C is correct. An asthma attack would put this child at the highest risk. Studies show that uncontrolled allergic rhinitis can make asthma much more challenging to manage. Since this child has a history of asthma, he should be treated aggressively to manage allergic rhinitis. Choices A, B, and D are incorrect. Although impaired sleep, decreased school performance, and irritability are all risks associated with allergic rhinitis, asthma is a life-threatening illness that must be treated quickly. Anything that interferes with that treatment (such as allergic rhinitis) should be avoided.

The nurse supervising a graduate nurse completes an incident report regarding a client who fell. Which of the following actions by the graduate nurse requires follow-up? Documents an objective description of what happened B. Indicates that a 2-inch laceration was present on the client's scalp C. Documents in the nursing note that an incident report was completed D. Notes the follow-up actions taken

Choice C is correct. An incident (sometimes termed an event or occurrence) report is completed for situations such as a client fall, medication error, visitor fall, assault from a visitor (or client), accidental discarding of client property, and a delay in care. The report is the facility's property and should not be mentioned in the client's medical record (including the nursing notes). If the record went to court, the incident/event report would be discoverable if mentioned in the client's medical record. Choices A, B, and D are incorrect. These are components of the incident report that should be included. The nurse should document objectively and include client statements and subjective and objective data. The nurse should also document interventions of an incident, such as contact with the physician.

The nurse administers dobutamine to a client with heart failure following a cardiac procedure. Which of the following is an intended effect of this medication? A. Increased heart rate B. Increased vasoconstriction C. Increased cardiac output D. Increased blood pressure

Choice C is correct. Dobutamine is a positive inotropic and chronotropic drug that helps increase myocardial contractility by selectively acting on the beta-1 receptors in the myocardium. By increasing the heart rate and contractility, dobutamine helps increase cardiac output in acute heart failure settings. Dobutamine is indicated in the short-term management of decompensated congestive heart failure.

he nurse is caring for a client with bulimia nervosa. The nurse anticipates a prescription for which medication? A. metformin B. bupropion C. fluoxetine D. clozapine

Choice C is correct. Fluoxetine is a selective serotonin reuptake inhibitor (SSRI) and is the only approved medication for bulimia nervosa. This medication effectively treats this disorder, especially when coupled with psychotherapy. Fluoxetine assists in increasing a client's weight and may mitigate comorbid disorders such as generalized anxiety disorder. Choice A, B, and D are incorrect. Metformin may cause weight loss and is quite advantageous in treating diabetes mellitus. Thus, this would not be an effective treatment for bulimia nervosa. Bupropion is an atypical antidepressant and causes weight loss. Bupropion is contraindicated in the treatment of bulimia because of its weight negative effects. Clozapine is an effective atypical antipsychotic that is not employed to treat bulimia. While some antipsychotics may be utilized for eating disorders, it is not common for an antipsychotic to be utilized for bulimia nervosa.

The 6-year-old immigrant child has been diagnosed with Hepatitis A. He was brought from Mexico by his grandparents a few days ago. You would expect that treatment for this child will include: A. Acyclovir B. Interferon C. Supportive care D. Ribavirin

Choice C is correct. Hepatitis A is typically an infection that is self-limiting if the child receives the appropriate supportive care. The disease is usually transmitted by drinking water and food that is contaminated with fecal matter. Removing the source of the infection and providing a healthy diet will often help resolve the infection. A hepatitis A vaccine is available that should be given to all children and high-risk adults. This vaccine should be given in two doses. Choice A is incorrect. Acyclovir is an antiviral that is given to slow the growth of the herpes virus. Choice B is incorrect. Interferon is a protein-based medication used in many immune system diseases such as multiple sclerosis (MS). Choice D is incorrect. Ribavirin is an antiviral medication used to treat hepatitis C, not hepatitis A.

Due to a recent flood, the only staff that were able to make it to work are two nursing assistants and one licensed practical nurse with the nurse manager. Knowing the different nursing delivery systems, which system should the nurse manager implement to care for the 20 clients admitted in their ward? A. Primary nursing B. Team nursing C. Functional nursing D. Case management

Choice C is correct. In functional nursing, each caregiver on a specific nursing unit is given specific tasks that fall into their scope of practice. In this situation, the nurse manager may administer medications to the entire group, while a licensed practical nurse performs treatments, and the client care attendants provide physical care. Choice A is incorrect. A registered nurse plans and organizes care for a group of clients and cares for this group during their entire hospitalization. This type of care delivery cannot be useful in this situation. Choice B is incorrect. An RN leads nursing staff who work together to provide care for a specific number of clients. The team typically consists of RNs, LPNs, and client care attendants. The team leader assesses client needs, plans client care, and revises the care plan based on changes in the client's condition. The leader assigns tasks to team members as needed. This cannot be done in this situation, as this requires too many staff members. Choice D is incorrect. Case management is a form of primary nursing that involves a registered nurse who manages the care of an assigned group of clients. This nurse coordinates care with the entire health care team. There is only one RN in the situation. Therefore, it cannot be used.

Which is an **intrinsic** risk factor that increases the risk of patients developing pressure ulcers? A. Shearing B. Friction C. Impaired tissue perfusion D. Pressure

Choice C is correct. Intrinsic refers to anything essential or belonging naturally. Impaired tissue perfusion is an internal risk factor. Other intrinsic risk factors associated with skin breakdown include: Poor nutritional status Incontinence Alterations in fluid balance Altered neurological functioning Choices A, B, and D are incorrect. Shearing, friction, and pressure are extrinsic (external) factors that increase the risk of impaired tissue performance that causes pressure sores.

The infection control nurse reviews guidelines with other nurses. Which of the following statements by the nurses would indicate a correct understanding of the teaching? . "The nurse should wear a surgical mask when transporting a client with active pulmonary tuberculosis (TB)." B. "Disposable utensils must be provided for a client infected with hepatitis B." C. "A surgical mask should be worn when working within three feet of the client infected with Neisseria meningitidis." D. "A surgical gown should be applied when entering a client's room with bacterial pneumonia."

Choice C is correct. Neisseria meningitidis is spread by infected droplets, and the nurse should wear a surgical mask while working within three feet of the client. Three feet is the distance for droplets to spread to another individual. If the client with Neisseria meningitidis should leave the room, they should wear a surgical mask. Choices A, B, and D are incorrect. When transporting a client with pulmonary tuberculosis, the client wears the surgical mask - not the nurse. Disposable dishes are not necessary to prevent the transmission of an infectious organism. The Centers for Disease Control and Prevention does not require disposable dishes for transmission-based precautions (contact, droplet, airborne). Bacterial pneumonia calls for droplet precautions (surgical mask), not a surgical gown, as that would be required for contact precautions.

The nurse is assessing a 9-month-old infant. Which of the following assessment findings would be expected? Select all that apply. Posterior fontanel has closed. Anterior fontanel is closed. Negative Moro reflex. Positive Babinski reflex. Sitting without support.

Choices A, C, D, and E are correct. These assessment findings are expected. The posterior fontanel closes between 2-3 months. The Moro (startle) reflex should be absent by 6 months. The Babinski (plantar) reflex should still be present until 12 months. The spinal column continues to straighten, and an infant sitting without support at 9 months is expected. The anterior fontanel should close at 18 months. In a 9-month-old infant, it should still be open.

The nurse is helping a client with a chest tube ambulate to the bathroom. The client turns suddenly and the chest tube becomes dislodged. What is the priority action for the nurse to take? A. Immediately re-insert the tube and call for help. B. Place your ungloved hand over the chest tube site and yell for help. C. Place a sterile dressing taped on three sides over the chest tube site and call for help. D. Monitor the patient's vital signs while he finishes ambulating to the bathroom and then call for help. Submit Answer

Choice C is correct. Placing a sterile occlusive dressing taped on three sides over the chest tube site and calling for help would be the priority actions. The nurse follows infection prevention by placing a sterile dressing over the site. Taping the dressing on three sides will cover the site. The open side will prevent a tension pneumothorax by allowing exhaled air to escape the dressing. The nurse should then immediately call for help.

The nurse is taking care of a client in the fourth stage of labor. She notes that her fundus is firm but she is still bleeding profusely. What should be the nurse's first action? A. Document the findings. B. Massage the client's fundus. C. Notify the physician. D. Put the client in Trendelenburg position.

Choice C is correct. Profuse bleeding may indicate a laceration of the birth canal or cervix, which needs the attention of a doctor to initiate appropriate interventions.

The nurse is teaching a parent of a 7-month-old client about food choices that may be introduced into the diet. The nurse should recommend which dietary item? A. cows milk B. apple juice C. soy-based yogurt D. flavored sports drinks

Choice C is correct. Soy-based yogurt is permitted as it does not contain added sugars and will not cause intestinal complications, unlike cow's milk products. Choices A, B, and D are incorrect. Cow's milk is prohibited until 12 months because it may put the infant at risk for intestinal and renal complications because of the dense amount of protein, sodium, and potassium. Because of the dense sugar, apple juice and flavored sports drinks are inappropriate for a 7-month-old. This type of juice should be introduced slowly, starting at 12 months.

Which of the following correctly describes the physical growth pattern from infancy into early childhood? A. Growth occurs from the distal to the proximal parts of the body B. Growth occurs from the proximal to the distal parts of the body C. Growth occurs in a head-to-toe progression D. Growth initially occurs most rapidly in the extremities

Choice C is correct. The cephalocaudal principle (also known as cephalocaudal development) refers to a general pattern of growth and development followed from infancy into toddlerhood and even early childhood whereby development follows a head-to-toe progression. Choice A is incorrect. Growth occurring from the distal to proximal portions of the body is not a recognized manner or pattern of physical growth or development for any age group, including those ranging from infancy into early childhood. Choice B is incorrect. Growth occurring from the proximal to the distal portions of the body is not a recognized manner or pattern of physical growth or development for any age group, including those ranging from infancy into early childhood. Choice D is incorrect. In children ranging in age from infancy to early childhood, growth does not initially occur most rapidly in the child's extremities. The general pattern of development occurs from the head downward through the body.

The nurse is planning to assist a respiratory therapist in performing a chest physiotherapy procedure. Which of the following is the initial action by the nurse before the process? A. Place a gown or fabric between the hands or percussion device and the client's skin B. Walk with the patient for a few laps around the unit to aid in percussion C. Administer a prescribed bronchodilator D. Call the physician to confirm x-ray results

Choice C is correct. The nurse should make sure that the patient receives a prescribed bronchodilator about 15 minutes before their chest physiotherapy procedure. Chest physiotherapy is used to loosen secretions trapped in the lungs. When administered before this procedure, a bronchodilator helps to dilate the bronchioles and liquify secretions.

If a female client weighed 7 lbs at birth, the nurse would expect her weight at her 2-year-old well-child visit to be: A. 35 lbs B. 40 lbs C. 21 lbs D. 28 lbs

Choice D is correct. A healthy child is expected to quadruple their weight by age 2.

The nurse is caring for a newborn immediately after delivery. Which of the following actions would be appropriate? A. Perform APGAR assessment at five and ten minutes B. Suctions the nose then the mouth C. Administer RhoGAM intramuscularly D. Place the infant skin to skin with a parent

Choice D is correct. A newborn is at risk of cold stress during the first few hours of post-intrauterine life. The nurse should dry the newborn thoroughly and place the newborn skin-to-skin with a parent. Choices A, B, and C are incorrect. The APGAR assessment is completed at one and five minutes. If suctioning is indicated, the nurse should suction the mouth, then the nose, not the nose, then the mouth. RhoGAM is indicated for Rh-negative mothers - not newborns. Medications administered to a newborn include intramuscular Vitamin K and erythromycin eye ointment.

The nurse is caring for a client who is receiving prescribed aspirin. Which of the following findings would indicate the client is having an adverse effect? A. Polyuria B. Hypokalemia C. Venous thromboembolism D. Black, tarry stools

Choice D is correct. Aspirin is an anti-inflammatory medication for acute myocardial infarction, rheumatic fever, and Kawasaki disease. The biggest concern with aspirin is its anticoagulant effects and ability to cause gastrointestinal bleeding associated with gastrointestinal ulcers. Choices A, B, and C are incorrect. Aspirin does not have a diuretic effect. The effect aspirin may have on the kidneys could be decreased renal blood flow, leading to renal insufficiency. Hypokalemia is not associated with aspirin usage. Venous thromboembolism is not associated with aspirin, considering the antiplatelet effect exerted by this medication.

The home health nurse is performing a follow-up visit on a child recently relocated to a new home following child abuse. The nurse anticipates that the child will likely demonstrate A. a willingness to explore new places. B. rapid social engagement with others. C. increased scholastic performance. D. reluctance or avoidance in social interactions.

Choice D is correct. Children subjected to abuse often have immediate psychosocial adverse effects such as acting aloof, being withdrawn, guarded, distrustful, and having difficulty engaging and maintaining social relationships.

Crisis helplines are highly important due to which of the following? Facilitates the ability of the nurse to visit the home. B. Allows the caller and the call center to plan follow-up care. C. Serves as a cost free way to develop new coping strategies. D. Often saves lives when a person is in a severe crisis.

Choice D is correct. Crisis helplines are highly relevant because these helplines often save lives when a person is in a severe crisis. These helplines are staffed with people who use somewhat scripted verbal communication that addresses the here and now of the crisis. The caller can remain anonymous if they choose to do so. They are also allowed to solve their immediate problem and be able to cope with their stressors in the crisis. Choice A is incorrect. Call centers for crises do not facilitate a visit to the person's home, but they do provide other needed help. When necessary, they may call for help to the person's house when the client's life is in danger. Choice B is incorrect. Although these call centers allow the caller and the call center to remain on the line, follow-up care is encouraged, but it is not planned. Choice C is incorrect. Crisis call lines are free of cost, but these lines are not intended to help the client to develop new coping strategies; this is done during the follow-up to the immediate crisis.

The nurse is assessing a client who is postoperative following a hypophysectomy. Which of the following findings should the nurse report to the primary healthcare provider (PHCP) immediately? A. Client reports a decreased smell B. No bowel movement in two days C. Foul-smelling breath D. Hourly urine output of 125 mL

Choice D is correct. Following hypophysectomy, the client is at risk of developing diabetes insipidus (DI). An hourly urine output of 125 mL would be considered polyuria because, in 24 hours, that would equate to 3000 mL. Choices A, B, and C are incorrect. A decreased smell is an expected finding after hypophysectomy. The client may experience this finding for a few months. Constipation is concerning because the client should be instructed not to strain for a bowel movement as this may increase intracranial pressure. The client is instructed not to brush their teeth following this procedure and should floss and use mouthwash as an alternative. Thus, incomplete oral care may produce foul-smelling breath.

The nurse has instructed a client newly diagnosed with the human immunodeficiency virus (HIV). Which of the following statements by the client would indicate effective understanding? This disease is caused by a retrovirus leading to A. encapsulation of CD4+ T-cells. B. inflammation of the CD4+ T-cells. C. abnormal proliferation of CD4+ T-cells. D. viral integration into the CD4+ T-cells.

Choice D is correct. HIV is a retrovirus because of its ability to insert itself into a cell's DNA via its viral RNA. This process causes the CD4/T-cell to be hijacked. HIV infection causes a virion to dock with a CD4/T-cell, which causes it to seize its nucleus. This hijacking alters the cell's DNA by inserting its viral RNA, which DNA then converts by an enzyme reverse transcriptase. This integrative process completes the process, making the CD4/T-cell able to create more HIV viral particles to infect other healthy CD4/T-cells.

A nurse is taking care of a client undergoing cerebral angiography. Which statement by the client would most warrant additional attention from the nurse? A. "I feel like I'm going to vomit." B. "I hope my results are okay." C. "It's getting a bit hot in here." D. "My throat is getting a bit itchy, and my eyes are getting watery."

Choice D is correct. Iodinated contrast materials are used during cerebral angiography, potentially causing severe allergic reactions. Here, the client's itchy throat and watery eyes are classic indications of an allergic reaction that may progress to an anaphylactic reaction. Symptoms of a severe anaphylactic reaction include airway compromise due to laryngeal edema or angioedema (stridor), bronchoconstriction (wheezing, cough, and dyspnea), and/or circulatory collapse (shock). This is an extreme emergency, as the client's airway is at risk of compromise. The nurse should promptly assess the client for additional signs of anaphylaxis, notify the health care provider (HCP), and initiate interventions to stop further symptom progression while alleviating the current manifestations. Choice A is incorrect. Nausea is likely a reaction to the administration of contrast material, presumably capable of being alleviated by the administration of a PRN (as needed) intravenous antiemetic medication. While this statement should be of concern to the nurse, this is not the priority concern. The nurse should prioritize airway-related symptoms in this client. Choice B is incorrect. This statement by the client may indicate anxiety or a flushing sensation being experienced by the client. The nurse should address the client's concern; however, it is not prioritized over an airway-related complaint. Choice C is incorrect. A warm, flushed feeling is a benign and anticipated reaction following the intravenous administration of contrast media. The sensation may be felt throughout the body but is often pronounced in certain regions - most clients experience this as a warm sensation around the throat, which gradually moves down to the pelvic area.

Minimizing and challenging the client's report of pain and pain intensity is: A. Often necessary if the client has a history of substance abuse. B. Often necessary if the client has a history of drug seeking behavior. C. Contrary to and in violation of the Nightingale oath. D. Contrary to and in violation of the American Nurses Association's standard of care. Submit Answer

Choice D is correct. Minimizing and challenging the client's report of pain/pain intensity is in violation of the American Nurses Association's standards of care about pain/pain management. Specifically, the American Nurses Association's Standards of Professional Performance for Pain Management Nursing. For example, nurses are mandated to document pain as expressed by the client regardless of what the nurse believes to be true and accurate.

The nurse is caring for a newborn immediately following birth. Which of the following actions by the nurse will prevent radiant heat loss in the newborn? A. Drying the newborns skin with a towel B. Placing the newborn on a padded, covered surface C. Using warmed, humidified oxygen D. Positioning the bassinet away from windows

Choice D is correct. Radiation is one of the mechanisms of heat loss. Radiant heat loss occurs when heat is transferred between two objects, not in direct contact. Radiant heat loss may occur if the infant is placed near windows, air conditioner vents, or drafts. Heat is transferred from a warmer to a cooler object -the infant's warmth is lost to the cooler object (e.g., windows/ cooler window glass). Radiant heat loss can occur despite the surrounding (ambient) air. Ambient will not fix this problem. The nurse must remove the source of the radiant heat loss, not adjust the air temperature. Choices A, B, and C are incorrect. Evaporative heat loss occurs when the moisture from the body's surface is lost to the environment. Drying the infant with a towel would help prevent evaporative heat loss, not radiant heat loss (choice A). Heat loss by conduction occurs when heat is transferred by direct contact from one object to the other. Placing the newborn on a padded, covered surface would prevent conductive heat loss (choice B). Convective heat loss happens when the heat is transferred from an object/ body to the surrounding air. Warmed, humidified oxygen can prevent convective heat loss (choice C).

A nurse cares for a client who has missed their last appointment with the primary healthcare provider (PHCP). The client states, "I missed my appointment because I overslept, but I knew it would be pointless anyway." The client is demonstrating which defense mechanism? A. Projection B. Reaction formation C. Denial D. Rationalization

Choice D is correct. Rationalization is a higher-level defense mechanism that involves an individual justifying behavior that is often offensive or abnormal through statements that they believe provide validation. However, rationalizing the behavior is done to avoid authentic feelings such as guilt if they have done something wrong. The client missing their appointment because they overslept is rationalizing this choice because they perceived the appointment as pointless. Choices A, B, and C are incorrect. The client is not demonstrating projection because they are not attributing their unacceptable feelings and thoughts to someone else. Reaction formation is when an individual acts opposite to their true feelings or actions. Denial is not exhibited because the client is acknowledging the missed appointment. Denial is when individuals block situations because they refuse to embrace the situation and associated emotions.

What are the expected fundal assessment findings for a woman who delivered a set of twins one hour ago via Cesarean section? A. The fundus is hard, midline, and 1-2 fingerbreadths above the umbilicus. B. The fundus need not be assessed because of the C-section. C. The fundus is to the right of the umbilicus and soft. D. The fundus is hard, midline, and at the level of the umbilicus.

Choice D is correct. Regardless of the mode of delivery, this is the normal postpartum fundus at one to two hours. Immediately after delivery and expulsion of the placenta, the uterus is about the size of a grapefruit and is located midline in the abdomen, halfway between the umbilicus and the symphysis pubis. Over the next several hours, the fundus will rise on the midline of the stomach to the level of or slightly above the umbilicus. Subsequently, the height of the fundus decreases by at least 1 cm or one fingerbreadth daily as the uterus goes through the process of involution. By the 10th day, the fundus is usually not palpable. Uterine "involution" refers to the return of the uterus to its pre-pregnancy size and condition. Involution begins soon after the expulsion of the placenta and occurs due to the contraction of the uterine smooth muscle. Assessing the fundus (top of the uterus) is a crucial component of post-delivery assessment—a lack of proper uterine involution results in complications such as postpartum hemorrhage. While assessing the fundus following delivery, you must evaluate for: Orientation: This represents the orientation of the fundus in relation to the umbilicus (referred by terms "to the left of the umbilicus", "midline", or "to the right of umbilicus"). Please note "midline" does not refer to the height of the fundus; instead, it refers to a side-to-side orientation in the midline (normal orientation). The fundus should immediately return to midline after delivery. If it deviates from the midline, it indicates a "distended bladder." Height: Assessing the height of the fundus is crucial to determine fundal descent because this is how the uterus returns to its original position in the true pelvis. Height is measured in fingerbreadths or centimeters (cm) in relation to the umbilicus (1 fingerbreadth = 1 to 1.5 centimeters). Please note immediately after delivery, the fundus is "below" the umbilicus- at midway between the symphysis pubis and umbilicus. The fundus then rises to the level of the umbilicus at about 1-2 hours post-delivery. At 12 hours, the fundus rises to 1 fingerbreadth above the umbilicus. Subsequently, it starts descending. In the next few days, the fundus "descends" at a

The nurse is discharging the client that has been admitted due to subarachnoid hemorrhage. The client still has some speech and balance deficits. Which referral should the nurse make? A. Refer the client to hospice care. B. Refer the client to speech therapy. C. Refer the client to physical therapy. D. Refer the client to a home health agency.

Choice D is correct. The client is going home, thus the client needs to be referred to a home health agency so that there is continuity of care even at home. Choice A is incorrect. Hospice care is for clients that are terminally ill. This client is not terminally ill. Choice B is incorrect. Speech therapy aids clients in regaining speech and swallowing abilities. Speech therapy should have been initiated and ongoing while the client was in the hospital. Choice C is incorrect. Physical therapy aids clients in regaining muscle strength and balance. Physical therapy should have been initiated and ongoing while the client was in the hospital.

A patient presents to the emergency department with pinpoint pupils, poor attention, and slurred speech. Upon assessment of vitals, the patient is found to have a BP of 92/60 mmHg, HR 58, RR 14, and T 96.8 degrees F. Which substance is this patient's intoxication most likely related to? A. Alcohol B. Cannabis C. Cocaine D. Opiates

Choice D is correct. The patient is showing signs of opiate intoxication. Opiate intoxication is characterized by pinpoint pupils, slurred speech, inattention, lethargy, psychomotor retardation, and impaired memory, judgment, and social function. Changes to vitals include hypotension, decreased heart rate, reduced temperature, and lower respiratory rate. Choice A is incorrect. Alcohol intoxication is characterized by unsteady gait/balance, nystagmus, flushed face, sedation, impaired judgment, uninhibited behavior, talkativeness, slurred speech, impaired memory, and irritability. Choice B is incorrect. Cannabis intoxication is characterized by reddened eyes, increased heart rate, dry mouth, hunger, loss of coordination/balance, relaxed mood, increased perceptions, social withdrawal, and paranoia. Choice C is incorrect. Cocaine intoxication is characterized by pupil dilation, increased or decreased heart rate and blood pressure, chills, nausea/vomiting, sweating, pacing, psychomotor agitation, visual or tactile hallucinations, and hyper-vigilance

While working in the Neonatal Intensive Care Unit (NICU), you are notified that a "small for gestational age" infant is being brought to the unit. Being a NICU nurse, you understand that this means which of the following? A. The infant's weight is less than 2500 grams. B. The infant's weight is below the 20th percentile. C. The infant's weight is less than 1500 grams. D. The infant's weight is below the 10th percentile.

Choice D is correct. The term "Small for Gestational Age (SGA)" is used when the infants are smaller than normal for the number of weeks of pregnancy (gestational age). When an infant's weight is below the 10th percentile for the gestational age, it is considered small for gestational age. By definition, about 10 percent of all newborns are labeled as SGA. Not all "Low Birth Weight" babies are SGA. Infants may be of low birth weight but may still fall above the 10th percentile for gestational age. It is important to distinguish SGA from other related terms, "Low Birth Weight (LBW)", "Very Low Birth Weight (VLBW), and "Extremely Low Birth Weight (ELBW)." These definitions are based on the infant's weight at the time of birth. These are not percentile scores and are defined on the absolute weight limit. An LBW infant is defined as an infant with a weight of less than 2500 grams (5 lb. and 8 ounces), regardless of gestational age at the time of birth. A VLBW infant is defined as one with a weight less than 1500 grams at the time of birth. An ELBW infant is less than 1000 grams at the time of birth.

The charge nurse is planning client care assignments for a registered nurse (RN) and licensed practical/vocational nurse (LPN/VN). Which of the following clients would be most appropriate to assign to the RN? A client with A. chronic anemia requiring epoetin injections. B. a resolving pneumothorax with a chest tube. C. a tracheostomy requiring intermittent suctioning. D. septic shock requiring intravenous (IV) vasopressors.

Choice D is correct. When making client assignments, the RN should be assigned the client with the least predictable outcome who is unstable. The client with septic shock receiving intravenous vasopressors should be assigned to the RN because of the need to titrate the vasopressors. Further, this client being in shock, is not stable and requires frequent assessment. Choices A, B, and C are incorrect. An LPN should be assigned clients who are stable and with a predictable outcome. A client with a chronic illness such as anemia requiring epoetin injections can be delegated to the LPN (Epoetin, unlike RhoGAM, is not regarded as a blood product). Further, the client with a resolving pneumothorax may be assigned to the LPN because the condition is resolving. Finally, LPNs may do suction in an established tracheostomy.

The nurse is assessing a client with venous thromboembolism in the lower extremity. Which of the following assessment findings would be expected? Select all that apply Pain Swelling Paralysis Pulse deficit Dependent rubor

Choices A and B are correct. Pain and swelling of the affected extremity are classic manifestations of venous thromboembolism. Other manifestations include warmth to the affected extremity and erythema. Choices C, D, and E are incorrect. Paralysis is a late manifestation associated with compartment syndrome not found in a VTE. A pulse deficit is a medical term describing the difference between the apical and peripheral pulse rates. This is a feature of an arrhythmia, not VTE. Dependent rubor is a classic manifestation associated with peripheral arterial disease.

The nurse is performing a physical assessment on a child with suspected Kawasaki disease (KD). Which of the following assessment findings would support this diagnosis? Select all that apply. strawberry tongue fruity breath drooling fever bright red rash on the cheeks

Choices A and D are correct. KD is an autoimmune disorder that occurs primarily in individuals younger than five. This condition may cause systemic vasculitis and cardiac abnormalities, including an aneurysm. The classic manifestations of KD include a high fever (unresponsive to antibiotics and antipyretics), red, cracked lips, strawberry tongue, and cervical lymphadenopathy. Choices B, C, and E are incorrect. Fruity breath is not a sign of Kawasaki's disease. Fruity breath is characteristic of diabetic ketoacidosis (DKA). Drooling is not a sign of Kawasaki's disease. Drooling is characteristic of a child presenting with epiglottitis. A bright red rash on the cheeks is a characteristic sign of Fifth's disease (parvovirus B19). The rash in KD is in the groin and perineum.

The nurse is assessing client who has a borderline personality disorder (BPD). Which of the following would be an expected finding? Self-mutilating behaviors Hypervigilance Emotional detachment Social inhibition Impulsivity

Choices A and E are correct. Borderline personality disorder (BPD) is a common personality disorder that features extreme emotional lability, impulsivity, self-mutilative behaviors, and manipulative mannerisms. Choices B, C, and D are incorrect. Hypervigilance is an expected finding with a paranoid personality disorder. Emotional detachment is compatible with a schizoid personality disorder. Social inhibition is consistent with an avoidant personality disorder.

The nurse is assessing a client with Dependent Personality Disorder. Which of the following would be an expected finding? Select all that apply. Difficulty with decision-making Flamboyant behaviors Intense and unstable relationships Avoiding social relationships Feels helpless when alone

Choices A and E are correct. Individuals with Dependent Personality Disorder manifest difficulty with decision-making and initiating projects. Choices B, C, and D are incorrect. Flamboyant behaviors and provocative dressing is a hallmark characteristics of histrionic personality disorder. Intense and unstable relationships are a cardinal manifestation of Borderline Personality Disorder. Finally, avoiding social relationships and situations is a finding with Avoidant Personality Disorder. Dependent Personality Disorder is a personality disorder characterized by difficulty with making decisions, problems with expressing disagreement, and often feeling helpless when alone. The nurse should encourage decision-making but never make decisions for the client.

A 30-year-old man comes into the clinic after being bitten by a wild skunk approximately 12 hours ago. The nurse knows that treatment for this client is likely to include: Select all that apply. Rabies immune globulin and vaccine Wound cleansing with povidone-iodine or saline solution and debridement Treatment with an appropriate antibiotic Suturing of the wound Debridement of wound edges Tetanus vaccine prophylactically

Choices A, B, C, E, and F are correct. Wild skunks have a high incidence of rabies and should be considered rabid. The patient should receive rabies immune globulin and vaccine. The CDC recommends the irrigation of the wound with povidone-iodine since that solution is virucidal and may help prevent infection. In the clinical judgment of the provider, saline can be safely substituted for povidone-iodine. Debridement of the wound edges may also help to prevent disease by cutting away tissue, clots, and other material in the wound. Any bite wound should be considered potentially infected, so an appropriate antibiotic and tetanus prophylaxis will be administered. Choice D is incorrect. The primary suturing of the injury is NOT recommended in this case. The bite is older than 8 hours and from an animal that has a high infection risk. Suturing the wound will close any potential infectious agents into the injury leading to an increased risk for infection.

The nurse is teaching a client who is breastfeeding and has developed mastitis. Which of the following statements by the nurse would be appropriate to make? Select all that apply. Continue to breastfeed your child normally Empty each breast at each feeding Complete the entire course of the prescribed antibiotic Wear a supportive bra without an underwire Wean breastfeeding during the infection

Choices A, B, C, and D are correct. It is essential to educate mothers with mastitis to continue breastfeeding. The infection will not be passed to their child, and they do not need to worry about adverse effects on their infants. The clogged milk ducts should become unclogged by continuing to breastfeed, and mastitis should improve. The client should be instructed to empty the breast completely at each feeding. Further, the prescribed antibiotic should be continued until it is gone (usually 7-10 days). Wearing a supportive bra but one without an underwire is appropriate educational advice for a mother with mastitis. The support will help with the pain and tenderness in the breasts, but an underwire could cause clogged milk ducts, so it should be avoided. Choice E is incorrect. Weaning an infant breastfeeding is inappropriate and contributes towards breastmilk engorgement and stasis. Breastfeeding or pumping every 1.5 to 2 hours makes the client comfortable and prevents stasis.

The nurse is caring for assigned clients. Which of the following actions would reflect effective care coordination? Select all that apply. Arranging for an interdisciplinary conference Consulting with case management for a discharge plan Initiating appropriate outpatient referrals Performing post-discharge phone calls Implementing transmission-based precautions

Choices A, B, C, and D are correct. The nurse arranging for an interdisciplinary conference, consulting with case management, initiating outpatient referrals, and performing post-discharge phone calls are all relevant to effective care coordination. These actions work to improve care delivery through effective communication with other members of the healthcare team. Effective care coordination involves providing organized, cost-effective care involving multiple disciplines through good communication. Effective care coordination aims to reduce costs by decreasing hospitalization and mitigating errors through linear communication.

The nurse is caring for a client with a port. Which of the following actions would be appropriate to take? Select all that apply. Access the port using sterile technique. Flush the port with heparin prior to de-access. Access the port using a 16-gauge catheter. Have the client wear a mask during the dressing change. Aspirate for blood return prior to medication administration.

Choices A, B, D, and E are correct. A port is a central venous line that is useful for individuals receiving chemotherapy. The nurse should utilize an aseptic technique to prevent central line-associated bloodstream infections (CLABSIs) when the port is accessed. This includes the nurse and the client wearing a mask as well as the nurse using sterile gloves. Occlusion is a common complication with a port, and prior to de-accessing, the nurse should flush heparin. Further, the client should be instructed to wear a mask to prevent contamination during dressing changes. Finally, the nurse must verify appropriate access by aspirating for blood return prior to medication administration. Choice C is incorrect. The nurse utilizes a non-coring needle to access a port. A 16-gauge catheter will be an option if a nurse starts a large-bore peripheral IV. When a port is accessed, it is accessed with a non-coring needle that is 0.5 to 2 inches, with the gauge being 19 to 22.

What findings are expected when assessing a patient with atelectasis? Select all that apply. Decreased breath sounds Decreased tactile fremitus Hyperresonance Shortness of breath Decreased oxygen saturation

Choices A, B, D, and E are correct. Incomplete lung expansion or the collapse of alveoli, known as atelectasis, prevents pressure changes and gas exchange by diffusion in the lungs. With atelectasis, lung tissue has collapsed, which leads to less lung mass available for oxygenation. The oxygen saturation is decreased, as well as breath sounds. Additionally, the patient will experience shortness of breath. Since alveoli collapse, there is more open space between the lung tissue and the chest wall. Open space does not transmit sound very well (decreased tactile fremitus). Areas of the lung with atelectasis cannot fulfill the function of respiration. Coughing, chest pain, cyanosis, dyspnea, and tachycardia are common symptoms of atelectasis. Choice C is incorrect. The percussion sound may be dull but not hyper-resonant in atelectasis or consolidation.

The nurse is caring for a child with eczema. Which of the following findings should the nurse expect? Select all that apply Erythema Pruritus Papules Skin ulcers Scaly circular rash

Choices A, B, and C are correct. Erythema is the superficial reddening of the skin. This redness is one of the most common symptoms of eczema and would be an expected assessment finding for all types of eczema. Pruritus is severe itching of the skin. Itching is one of the most common symptoms of eczema and would be an expected assessment finding for all types of eczema. Papules are solid elevations of skin with no visible fluid less than 1 cm in diameter. Although not all patients with eczema will necessarily have papules, they are a common assessment finding. Choices D and E are incorrect. Skin ulcers are round sores that develop because of a lack of oxygen-rich blood flow. This can be caused by excessive pressure placed on the skin. A scaly circular rash is a common manifestation associated with a fungal infection. This is known as ringworm. This contagious infection may affect the skin and the scalp.

The nurse is reviewing nonreassuring fetal heart rate patterns with a group of students. It would indicate effective understanding if the student identifies which pattern as nonreassuring? Select all that apply. fetal bradycardia variable decelerations late decelerations early decelerations accelerations

Choices A, B, and C are correct. Fetal bradycardia, or a decrease in fetal heart rate below 110 bpm, is a non-reassuring sign on a fetal heart rate strip. When the nurse notes this sign, the nurse must intervene by repositioning the mother on her left side, increasing IV fluids, administering oxygen, and notifying the healthcare provider quickly. Also, fetal bradycardia is often a result of uterine hyperstimulation. The nurse should discontinue the infusion if the client is on an oxytocin drip. Variable decelerations, or sharp and profound drops in the fetal heart rate unrelated to the time of contractions, are a non-reassuring sign on a fetal heart rate strip. Anytime the nurse notes this sign, intervention is necessary by lying the mother on her left side, increasing IV fluids, administering oxygen, and notifying the healthcare provider quickly. Variable decelerations are often caused by cord compression, such as a prolapsed cord, and would be an emergency requiring quick nursing intervention. Late decelerations, or dips in the fetal heart rate after a contraction, are a non-reassuring sign on a fetal heart rate strip. Anytime the nurse notes late decelerations, the nurse should lay the client on her left side, increase IV fluids, administer oxygen via face mask, and notify the healthcare provider quickly. Late decelerations are due to uteroplacental insufficiency and require intervention by the nurse. Nonreassuring fetal heart patterns include Bradycardia Tachycardia Late decelerations Prolonged decelerations Hypertonic uterine activity Variable decelerations (less than 70 beats per minute longer than 60 seconds)

The nurse manager is completing an annual performance evaluation of a staff nurse. Which elements should the nurse manager include when completing the evaluation? Select all that apply. The nurses' bar-code medication administration scan rate The number of times the nurse has been absent or tardy The nurse achieving a national certification The nurses' performance compared to other staff nurses The number of medication errors the nurse has self-reported.

Choices A, B, and C are correct. The performance appraisal/evaluation goal is to provide a broad review of the employee's performance with minimal evaluator bias. The more objective the evaluation, the less the bias. Objective metrics such as bar-code medication administration rate, attendance, and national certifications are logical elements to include in the appraisal. Choices D and E are incorrect. The nurses' performance should not be compared/contrasted with other nurses. The annual performance review should be focused solely on the nurses' performance. Self-reporting is valued by the nursing profession and promotes a culture of safety. Using self-reports of a medication error against the nurses' performance would likely discourage future reporting. If the nurse manager observes unsafe practices by the nurse, they should be corrected. However, self-reporting should be encouraged and not weaponized against the nurse.

The nurse is conducting a health screening at a local health fair. Which of the following should the nurse recognize as a risk factor for developing testicular cancer? Select all that apply. Cryptorchidism Human immunodeficiency virus (HIV) Vasectomy Family history Herpes simplex virus (HSV)

Choices A, B, and D are correct. Risk factors for testicular cancer include cryptorchidism, human immunodeficiency virus (HIV), and family history. Cryptorchidism (Choice A) refers to undescended testicle where the testicle fails to descend to its normal position in the scrotum. Undescended testicles are associated with decreased fertility, testicular torsion, inguinal hernias, and an increased risk of testicular germ cell tumors. HIV-positive ( Choice B) men are more likely to develop testicular cancer. Family history (Choice D) of testicular cancer is another risk factor, with 8-10 times increased risk if the man has a sibling with testicular cancer. Choices C and E are incorrect. A vasectomy is not a risk factor for testicular cancer. This procedure is done to interrupt each vas deferens to induce male sterility. Herpes Simplex Virus (HSV) is a virus that may be transmitted sexually but does not raise the risk for testicular cancer, unlike HIV.

The nurse has provided medication instruction to a client prescribed sucralfate. Which of the following statements, if made by the client, would require further teaching? Select all that apply. I should take this medication one hour after meals." "I will remain upright for 30 minutes after taking this medicine." "This medication will help with my peptic ulcer disease." "I know this medication works when my nausea and vomiting are gone." "I may dissolve this medication in warm water."

Choices A, B, and D are correct. These statements require further teaching. Sucralfate is a medication indicated in peptic ulcer disease. This medication should be taken one hour before meals as the medication will coat the ulcer allowing a client to eat meals without pain. The client is not required to be upright 30 minutes after taking this medication. This would be applicable instruction for a patient prescribed a bisphosphonate. This medication has no indication for nausea and vomiting treatment. Appropriate treatment for nausea and vomiting would be ondansetron or metoclopramide.

The nurse is teaching a group of students about incident reports. Which of the following situations would require an incident report? A visitor Select all that apply. refusing to wear personal protective equipment (PPE). adjusting a client's infusion pump. requesting that their family member get pain medication. assisting their family member with brushing their teeth. stating that they fell while using the bathroom.

Choices A, B, and E are correct. Incident (sometimes termed occurrence or event) reporting is required when any activity deviates from the norm. Incident reporting may be completed for visitors. Events that warrant reporting would include the refusal to wear PPE, tampering with medical devices (such as adjusting an infusion pump), and reporting that they fell while using the bathroom.

The nurse is developing a plan of care for a 9-year-old child with a vaso-occlusive crisis (VOC) related to sickle cell anemia (SCA). Which of the following should the nurse include in the child's plan of care? Select all that apply. Obtain a prescription for patient-controlled analgesia Perform frequent pain assessments Provide ice packs to the client's joints Prepare to infuse hypertonic intravenous fluids Plan activities with frequent rest periods Anticipate an order to transfusion fresh frozen plasma

Choices A, B, and E are correct. Obtaining a prescription for a PCA is an effective intervention for a client experiencing a vaso-occlusive crisis (VOC) because PCAs give them autonomy in their pain control. PCAs can be used as early as age 7. Opioids are preferred in managing pain for a client with a VOC and are advantageous because they can be administered intravenously, orally, or intranasally. Effective pain control is the cornerstone of the management of a VOC. This, coupled with frequent pain assessments, are essential nursing actions. Additionally, the anemia may cause the child to experience fatigue. The nurse should plan activities with frequent rest breaks because, during a VOC, the client experiences significant fatigue. Also, excessive physical activity may make the pain worse. Choices C, D, and F are incorrect. Ice and cold compresses should not be used during a VOC because they may make the pain worse because of the vasoconstriction effects. Hydration is a key priority in a client with a VOC. However, hypertonic IV fluids are not utilized. A short course of hypotonic fluids (0.45% saline) and isotonic saline is primarily used during a VOC. Packed red blood cells may be ordered for a client with significant anemia to increase hemoglobin levels. FFP would not be utilized as it helps provide clotting factors.

The nurse is teaching a group of students about using reminiscence therapy. Which statements should the nurse include in the teaching? Select all that apply. This approach helps support self-esteem This is an effective intervention in a group setting This intervention focuses on looking forward Establishing future goals is important part of this intervention Reminiscing is a way to express personal identity

Choices A, B, and E are correct. Reminiscence helps support self-esteem by having an individual look back on past accomplishments and positive life experiences. This strategy may be used one-on-one or in a group setting, facilitating rapport building with other individuals. Finally, reminiscence is a way for an individual to express their personal identity by reflecting on past accomplishments (college work, occupations, marriage, etc.).

The nurse conducts safety rounds within the nursing unit. Which observation requires follow-up? Select all that apply. The client's armband was affixed to the bedside table. The client's telephone number and name were used as identifiers. Multiple blood specimen tubes are labeled before specimen collection. A room number is used as an identifier during medication administration. Verifies client's name, date of birth, consent, site, and procedure during a time out process.

Choices A, C, and D are correct. These observations require follow-up because they are unsafe. The client's armband should not be the sole source of client identification but rather collateral information. It should be attached to the client, not a bedside table or bed. Specimen tubes should never be prelabeled. Specimen collection requires appropriate labeling and client verification when obtaining the specimen. A room number cannot be used as a reliable client identifier. Choices B and E are incorrect. These observations reflect appropriate practice and do not require follow-up. A client's telephone number is an acceptable identifier when combined with their name. A time-out procedure should involve the client and participants of the procedure. Elements include the client's name, date of birth, consent, site, and procedure during a time-out process.

Which of the following nursing improvements follow the recommendations of the Institute of Medicine's Committee on Quality Healthcare in America? Basing patient care on continuous healing relationships Customizing care to reflect the competencies of the staff Using evidence-based decision making Having a charge nurse as the source of control Using safety as a system priority Recognizing the need for secrecy to protect patient privacy

Choices A, C, and E are correct. Standards are the levels of performance accepted and expected by the nursing staff or other healthcare team members. They are established by authority, custom, or consent. The Committee on Quality Health Care in America of the Institute of Medicine, in its report Crossing the Quality Chasm, highlights six aims to be met by health care systems about quality care: Safe: Avoiding injury Useful: Avoiding overuse and underuse Patient-centered: Responding to patient preferences, needs, and values Timely: Reducing waits and delays Efficient: Avoiding waste Equitable: Providing care that does not vary in quality to all recipients

Which of the following statements best describe the stage of development that should be expected of preschoolers based on developmental theories? Select all that apply. According to Freud, the child is in the phallic stage. According to Erikson, the child is in the trust vs. mistrust stage. According to Havighurst, the child is learning to get along with others. According to Fowler, the child imitates the religious behavior of others. According to Kohlberg, the child defines satisfying acts as right. According to Havighurst, the child is achieving gender-specific roles

Choices A, D, and E are correct. According to Freud, the child is in the phallic stage. According to Fowler, the child imitates the religious behavior of others. According to Kohlberg, the child defines satisfying acts as right. Human growth and development have been studied since the beginning of the 20th century and many theories have been developed to explain human responses usually occurring at certain ages during life. Although a psychological approach is familiar to most developmental theories, each method has a different focus. Different opinions examine cognitive, social, and instinctual influences on human growth and development. Choice B is incorrect. According to Erikson, the child is in the initiative versus guilt stage. Choice C is incorrect. According to Havighurts, the child is learning sex differences, forming concepts, and getting ready to read. Choice F is incorrect. According to Havighurst, the adolescent, not the preschooler, is achieving gender-specific social roles.

The nurse is caring for a client with a tracheostomy receiving oxygen via tracheostomy collar. The nurse should plan to Select all that apply. (select 2) Plan to suction the tracheostomy every two to four hours Ensure that the oxygen is humidified Instill normal saline down the tracheostomy immediately before suctioning Suction the tracheostomy for a maximum of three passes Apply suction as the catheter is inserted into the tracheostomy

Choices B and D are correct. When caring for a client with a tracheostomy, suctioning should be performed only when clinically indicated. Indications for suctioning the client include tachypnea, rhonchi in the lung fields, and decreasing oxygen saturation. When suctioning a tracheostomy, the nurse should use a sterile technique, and a maximum of three passes should be completed. The oxygen must be warm and humidified for a client receiving oxygen via a trach collar. If it is not appropriately warmed or humidified, tracheal damage may occur. The humification assists with the passage of the secretions. Choices A, C, and E are incorrect. Instilling saline down the tracheostomy prior to suctioning has shown little benefit and may be harmful because it could lead to pneumonia. When the client's tracheostomy is suctioned, it should be suctioned as the catheter is withdrawn for 10-15 seconds. The suctioning should be continuous so secretions are not dropped back down the airway. Suctioning should not be scheduled; instead, it should be based on the client's respiratory assessment.

You are monitoring an 18-month old child who has just had surgical correction of an epispadias completed. Which of the following assessment findings would the nurse need to report to the healthcare provider? Select all that apply. Urine output of 2.2 mL/Kg/hr. Cloudy, foul smelling urine. Stent in the meatus appears clogged. Temperature of 37.4 degrees Celsius. Resting heart rate of 180 bpm

Choices B, C and E are correct. Cloudy, foul-smelling urine could indicate an infection and needs to be reported to the healthcare provider. If the stent placed in the urethral meatus appears clogged, no urine will be able to exit the bladder. This needs to be corrected surgically, and the provider needs to be notified. A heart rate of 180 bpm in an 18-month old child is considered tachycardia. The child may be in pain or could be developing an infection, which needs to be reported to the healthcare provider. Choice A is incorrect. Urine output greater than 1.5 mL/Kg/hr is desired in children, so this is a normal finding. If there had been no urine output for 1 hour after surgery, the nurse would need to notify the provider. Choice D is incorrect. A temperature of 36.4 to 37.4 degrees Celsius is average. A child is not considered febrile until their temperature reaches 38.0 degrees Celsius.

What health issues might you expect to find in a client that is a victim of domestic violence? Select all that apply. Upper respiratory infections Bruises and broken bones Unintended pregnancies Repetitive strain injuries Alcoholism Depression

Choices B, C, E, and F are correct. Domestic violence (including physical, emotional, and sexual abuse) occurs throughout society. It is present among all racial, social, and economic groups. Health issues related to domestic violence include physical injury from the assault and chronic health problems that may emerge, either as a complication of traumatic injury or as a physical response to ongoing stress from violence or neglect. Health issues related to domestic violence include physical injury from the assault itself, such as bruises and broken bones (Choice B). Families experiencing domestic violence/ physical abuse have more unintended pregnancies, miscarriages, abortions, and low-birth-weight babies (Choice C). Families experiencing domestic violence have higher rates of substance abuse and depression (Choices E and F). Choice A is incorrect. While stress may affect immunity, upper respiratory infections are not particularly associated with physical abuse. Choice D is incorrect. Repetitive strain injuries are not particularly associated with physical abuse. They are seen with repetitive tasks performed over long periods, such as typing and using a computer mouse or assembling parts in a factory line

The nurse is educating staff on adult basic life support. It would be appropriate to include which of the following? Select all that apply. Carotid pulse check should not take more than 20 seconds. The rate of chest compressions should be 100-120 per minute. Chest compression depth should be 2 inches on the center breastbone. Chest tube insertion should be prepared after five minutes of CPR. Early defibrillation is essential in the survival of ventricular fibrillation.

Choices B, C, and E are correct. High-quality CPR involves a compression depth of two inches on the center breastbone. The rate of the compressions should be 100-120 per minute. The nurse should utilize early defibrillation as it is the most effective treatment for ventricular fibrillation. Choices A and D are incorrect. A carotid pulse check should not exceed ten seconds. Poor quality CPR has been linked to prolonged pulse checks and pulse checks occurring too frequently. A chest tube is not in the BLS algorithm and would only be utilized for chest trauma causing injuries such as pneumothorax.

A client is seeking guidance on secondary prevention strategies to prevent cancer. Which strategies would be most appropriate for the nurse to include in the client's educational plan? Select all that apply. Eliminate alcohol intake Pap smears Rehabilitation programs Colonoscopies Regular cancer screenings

Choices B, D, and E are correct. Secondary prevention strategies use screening tests that aim to detect cancer at an early stage. This makes it possible to treat cancer early and increases the chance of a cure. A pap smear is a secondary prevention strategy because it is a screening test to detect cervical cancer early (Choice B). Colonoscopy is a secondary prevention strategy because it detects pre-cancerous colon polyps before they progress to colon cancer (Choice D). Regular screenings, such as mammograms, colonoscopies, and pap smears, can help detect cancer early when it's most treatable. The specific screening recommendations vary depending on age, sex, and other factors, so it's important to discuss screening options with a healthcare provider (Choice E). Choices A and C are incorrect. Alcohol is implicated as a carcinogen in a variety of cancers. Eliminating alcohol is a primary prevention strategy (Choice A). Rehabilitation programs are a tertiary prevention strategy (Choice C).

The nurse is observing infection control practices in the nursing unit. Which of the following findings require follow-up? A client with Select all that apply. H. pylori placed on standard precautions. rotavirus provided a disposable blood pressure cuff. rubella and their door is kept closed. influenza ambulating in the hall with a surgical mask. Legionnaires' disease placed on contact precautions.

Choices C and E are correct. These observations are inappropriate and require follow-up. The door should be closed in airborne isolation precautions, not droplet precautions. A client with rubella should be placed on droplet precautions. The minimum PPE required for droplet precautions is a surgical mask. Legionnaires' disease is not transmitted person-to-person but rather through infected water or soil. This bacterium requires standard precautions.

The pediatric nurse is treating an 18-month-old who has tested positive for Respiratory Syncytial Virus (RSV). Which of the following signs and symptoms would the nurse expect to find? Select all that apply. Thin nasal secretions. Productive cough. Bradypnea. Nasal flaring. Crackles in lungs. Drooling.

Explanation Choices A, D, & E are correct. A is correct. Thin nasal secretions are an expected symptom of RSV. This is an acute viral infection that affects the bronchioles. Children experience significant upper respiratory congestion when dealing with RSV and may need frequent suctioning to keep the airway clear and lessen the work of breathing. D is correct. Nasal flaring is an expected sign of RSV. This is a typical signal of respiratory distress in an infant or young child. The nares flare outward with inspiration due to use of accessory muscles and increased effort to breathe. E is correct. Crackles in the lungs are an expected finding with RSV. This sound occurs due to inflammation or fluid accumulation in the alveoli which results in decreased gas exchange. Wheezing may also occur due to the thick mucus in the bronchioles that restricts air flow Choice B is incorrect. The cough found with RSV is typically nonproductive. Upon auscultation, the nurse may note wheezing in the lungs and other signs of increased work of breathing. Their cough will sound dry and be spontaneous, but it does not typically produce any sputum. Choice C is incorrect. Bradypnea is not an expected finding of RSV. Instead, one would expect to see tachypnea. In children, we typically see vital sign numbers go up before they go down due to compensation. A child with RSV must work harder to breathe and keep their body oxygenated and will breathe faster to try to keep up.

The nurse is providing discharge instructions to a client prescribed lisinopril. Which of the following instructions should the nurse include? Select all that apply. You will need to take your pulse for one minute before each dose. You may notice the need to go to the bathroom more often. Limit your intake of foods such as avocados and apricots. You may notice a decrease in your ability to taste foods. The goal of this medication is to lower your cholesterol.

Lisinopril is an ACE inhibitor (ACE-I) and may cause hyperkalemia. It would be correct for the nurse to instruct the client to limit their intake of potassium-rich foods such as avocados, bananas, apricots, and legumes. If the client lowers their potassium intake, it could decrease the likelihood of developing dangerously high potassium levels. ACEIs may also cause a client to have a decreased taste sensation (dysgeusia). This may make the client more likely to use salt, worsening hypertension. Thus, it is appropriate to instruct the client to season their food more naturally.

he following scenario applies to the next 1 items A 12-year-old is evaluated for generalized pain in the emergency department (ED) A 12-year-old male with sickle cell anemia presents with his father with generalized pain that worsened overnight. The client stated his pain is in his knees and ankles. His knees and ankles had a full range of motion on exam and were tender to the touch. Slight swelling in the ankles and knees was observed. The client states that the pain 'hurts a lot and he had difficulty sleeping with the pain. He rates his pain as a '7' on a scale from 0-10. The father denies any recent illnesses or any strenuous activity. The child's vaccines are current, and he takes no medication daily. orders: Establish peripheral vascular access Start patient-controlled analgesia, intravenous (IV) morphine 250 mL of 0.9% saline over one-hour Complete blood count (CBC); Complete metabolic panel (CMP) vital signs: Temperature 98.0o F (37o C) Pulse 93/minute Respirations 16/minute Blood Pressure 113/81 mm Hg Oxygen saturation 96% on room air The nurse is implementing care for this client ➢ Click to specify if each nursing action is appropriate or not appropriate Have a second nurse

Regardless of the prescribed PCA setting, the nurse should verify the settings with a second nurse as the pump is being initiated. This helps reduce errors with morphine which is a high-risk medication. These settings should also be verified during the handoff report and any time prescribed changes are made. It would be appropriate for the nurse to apply EMLA (topical lidocaine) via a disk or thick cream to the skin prior to starting an IV can decrease the discomfort associated with the procedure. This cream is applied 30 to 60 minutes prior to starting the IV. It would not be appropriate for the nurse to administer oxygen via nasal cannula as this client has an optimal oxygen saturation. It is detrimental to administer supplemental oxygen to a client with SCA who is appropriately oxygenating. Oxygen does not reverse sickled RBCs, and if used in a nonhypoxic patient, it decreases erythropoiesis. It is not necessary to keep the client NPO. It would be beneficial to encourage the client to increase their intake of fluids and fluid rich foods such as fruit. Finally, warm compresses effectively provide pain relief because of the vasodilation effect. It does not reverse sickled RBCs, and if used in a nonhypoxic patient, it decreases erythropoiesis.

A 63-year-old male presents with concerns about difficulty with driving at nighttime. The client states, 'I feel like my vision is blurred, and I cannot change it.' The client reports no pain in either eye. He reports that this problem has 'gotten worse and cannot drive anymore because it is hard to see at night' and it started 'some time ago' and cannot pinpoint an exact start. The assessment showed the pupils were equal, round, and briskly reactive to light. They measured 3 mm. Slight opacity was noted in both eyes. No loss in the visual fields. Diagnostic: tonometry 20 mmHg out of 10 -21 mmhg The nurse reviews the assessment findings ➢ Click to specify if the assessment findings are consistent with cataracts or glaucoma. Each finding may support more than one disease process Vision impairment worse at nighttime Opacity in the eye Blurred vision Increase in intraocular pressure (IOP)

The client reported vision impairment worse at nighttime is a classic manifestation of cataracts. This often is the symptom that triggers a client to seek medical care because it makes driving difficult. A client with cataracts has a problem with the eye lens where opacities form. Cataracts may impact one or both eyes. Blurred vision is a manifestation of both cataracts and glaucoma. Glaucoma may cause a loss in peripheral vision. Ocular pressure may be measured by tonometry and is often higher in the morning when compared to the afternoon and evening hours. Cataracts are a problem with the lens, whereas glaucoma is a problem with increased intraocular pressure. The increase in IOP is classically associated with glaucoma Additional Info Cataracts: Cataracts may develop as an individual ages. Factors that increase cataracts include UV exposure. Trouble with discriminating colors and nighttime driving is common. Blurred vision that feels 'smudged.' Age-related cataracts do not cause pain. Glaucoma: Glaucoma is caused by increased intraocular pressure (greater than 21 mm Hg). Vision may be blurred, and may have peripheral vision loss. Headache and brow pain may be endorsed. Primary angle-closure glaucoma (PACG) or acute glaucoma has a sudden onset and is an emergency.

The nurse is assessing a client who just returned from surgery. The nurse checks preoperative vital signs at 8:30 AM to compare them with the current vital signs at 10:30 AM. What action should the nurse take? 8:30: 70 bpm 16RR 110/65 bp 95.5t 92% o2 10:30: 101 BPM 18RR 90/52BP 99.2T 96%o2 A. Assess the surgical wound B. Collect blood cultures C. Administer oxygen at 2 L/minute D. Encourage by-mouth (PO) fluids

The client's 10:30 AM vital signs show signs of shock. Considering this client is in the immediate postoperative period, the nurse should assess the surgical wound for signs of hemorrhage. The nurse should reinforce the dressing if this is the source of the bleeding. The nurse should notify the primary healthcare physician (PHCP) of the client's change in condition. ✓ Immediately post-operatively, the client will likely experience a fluid volume deficit. ✓ This deficit is likely from the client being NPO preoperatively and losing blood and bodily fluids during the procedure. ✓ The client is at risk for hypovolemic shock if the fluid imbalance is not corrected. ✓ The earliest signs of hypovolemic shock includes tachycardia, restlessness, and altered mental status.

What is the correct documentation of the patient's peripheral pulse when the finding is that the posterior tibial pulse is weak and thready? A. Grade C posterior tibial pulse B. Posterior tibial pulse is Grade B C. The client's posterior tibial is 2 D. Posterior tibial pulse is 1

When assessing pulses, the strength, volume, and fullness of the peripheral pulses are categorized and documented as follows: 0: Absent pulses 1: Weak pulse 2: Normal pulse 3: Increased volume 4: Abounding pulse Choices A and B are incorrect. Grades and grading are not used to document pulses. Choice C is incorrect. The pulse is weak and thready, not regular.

You are a nurse in the local childcare facility. You are feeding an infant from a bottle containing expressed breast milk from the mother, halfway through the feeding, you realize that the breastmilk you are supplying is not for this child. You have mistakenly picked up the breastmilk that was for another woman's child. You should: Select all that apply. Inform the parent of the child that you are feeding. Inform the mother of the child whose milk you fed to the child. Complete an incident report per facility policy. Inform the providers who are caring for the infants. Take steps to prevent future errors

all are correct Choices A, B, C, D and E are correct. All of these actions are appropriate and expected in this situation. Also, the team should assess both of the mothers for any infectious process. Additionally, the nurse should educate both sets of parents that the risk of transmission of the disease is small. The mother may have concerns about exposure to hepatitis B and C; however, these infections cannot be spread from a woman to an infant through breastmilk. Probably the most critical intervention is to put processes in place to prevent mix-ups of milk from happening again. The healthcare facility should review the incident (Choice E) and take steps to prevent similar errors from occurring in the future, such as staff education, process improvements, or implementing technology to reduce medication errors. If breast milk is fed to the wrong baby, there is a risk of transmission of infectious agents from the donor to the recipient. ✓ According to the Academy of Breastfeeding Medicine, the most common infectious agents that may be transmitted through breast milk include bacteria, viruses, and fungi. Staphylococcus aureus Streptococcus, and Escherichia coli (E. coli) Viral contaminants may include cytomegalovirus (CMV) Human immunodeficiency virus (HIV) Hepatitis viruses Fungal infections, such as candidiasis, can also be transmitted through breast milk. ✓ In addition to infectious agents, other types of contaminants that may be present in breast milk include medications, drugs of abuse, and environmental toxins . ✓ For example, if the donor is taking medication for a medical condition, the medication may be present in the breast milk and may cause adverse effects in the recipient if given inappropriately. ✓ It is important to note that these contaminants are not always present in breast milk, and the risk of transmission depends on a variety of factors, including the health status of the donor and recipient, the presence of any underlying medical conditions, and the handling and storage of the breast milk

The nurse just finished receiving the shift report from the night nurse. Which of the following newborns should the nurse assess first? A. A 3-hour old newborn weighing 6 pounds B. A 4-hour old newborn delivered at 42 weeks C. A 6-hour old newborn that is 21 inches long D. An 8-hour old newborn delivered at 40 weeks

choice B is correct. Post-maturity refers to any baby born at or beyond 42 weeks gestation (42 0/7 weeks) or at or beyond 294 days from the first day of the mother's last menstrual period (LMP). Post-maturity is also referred to as prolonged pregnancy, post-term, and post-dates pregnancy. At about 40-42 weeks, placental insufficiency ensues due to the aging placenta. Therefore, the infants rely on their subcutaneous fat reserves to sustain them after 40 to 42 weeks since the aging placenta is unable to provide the necessary nutrition. Due to these depleted subcutaneous fat reserves, the post-term infant is at risk for hypoglycemia and hypothermia. In at-risk infants, the incidence of neonatal hypoglycemia is highest in the first few hours after birth. In this case (Choice B), a 4-hour old infant delivered at 42 weeks is at-risk. Additionally, the risk of meconium aspiration is high in the post-term fetuses and can cause respiratory distress when the baby is born. The nurse should prioritize and assess this post-term infant first. Choice A is incorrect. According to the World Health Organization (WHO), the average birth weight for a full-term baby is around 7.5 lb. However, a birth weight range between 5.5 lb. (2.5 kg) and 8.2 lb. (4.0 kg) is considered normal. Small for gestational age (SGA) is defined as a birth weight of less than 10th percentile for gestational age. Large for gestational age (LGA) refers to a birth weight equal to or more than the 90th percentile for a given gestational age. Macrosomia refers to a birth weight greater than 4000 to 4500 grams ( 4 to 4.5 kg), regardless of gestational age. The infant weighing 6 pounds (Choice A) is within the normal weight range for a newborn; the nurse does not need to see this infant first. Choice C is incorrect. The average length of full-term babies at birth is 20 inches (50 cm). The normal range is between 18 to 22 inches. Macrosomia is defined based on the birth weight, not the birth length. Twenty-one inches is a bit longer than the average for most infants, but this is not a relevant finding that causes complications soon after birth. The nurse does not need to see this infant first. Choice D is incorrect. The risk of neonatal hypoglycemia is highest

The nurse is educating a new graduate nurse on different forms of therapeutic communication. Select the form of therapeutic communication which is accurately paired with the correct description of that form of therapeutic communication. A. Seeking Clarification: Utilizing open-ended questions rather than closed-ended questions B. Offering General Leads: Ensuring the client fully understands the sent message C. Reflection: By repeating the client's words back to the client, the nurse conveys that they are actively listening while concurrently encouraging further expression from the client D. Offering Self: Giving the client advice based on the opinion of the nurse

hoice C is correct. Reflection is a way of helping clients better understand their thoughts and feelings. By utilizing a question, simple statement, or the client's own words, the nurse may convey their observations of the client. Choice A is incorrect. Utilizing a seeking clarification therapeutic communication technique does not limit a nurse to only using open-ended questions, nor forbid them from using closed-ended questions. The goal of this therapeutic communication technique is to make clear that which is vague or maximize understanding between the nurse and client. A number of clarification techniques (i.e., paraphrasing, open-ended questions, closed-ended questions, reflecting, restating, etc.) may be used to achieve this goal when seeking clarification. Choice B is incorrect. When offering general leads, the nurse allows the client to take direction in the discussion. By using phrases such as "go on" or "and then?" the nurse conveys interest in what occurs next in the client's story. Choice D is incorrect. Hospital stays can be lonely and stressful at times. When nurses are present with their clients, it shows clients that the nurses value them and are willing to provide them time and attention. The mere offering by a nurse to be present with a client for a few minutes is a powerful way to create a caring connection.

A nurse at a community clinic is taking care of a 34-year-old patient who has been prescribed oral prednisone to treat respiratory issues. What education should the nurse include concerning the possible side effects of this medication? Select all that apply. Increased susceptibility to infection Weight gain Insomnia Blood glucose elevation Increased urine output

hoices A, B, C, and D are correct. A is correct. Prednisone is a corticosteroid that suppresses the immune system, making the patient more susceptible to infections. B is correct. Prednisone can increase appetite, which can lead to weight gain. It can also cause fluid retention and redistribution of fat, contributing to weight gain. C is correct. Prednisone can affect the sleep-wake cycle, causing difficulty sleeping or insomnia. D is correct. Prednisone can raise blood sugar levels, potentially leading to or exacerbating diabetes. Choice E is incorrect. E is incorrect. Prednisone does not typically increase urine output. This is more commonly associated with diuretics. Prednisone can, however, cause fluid retention. Prednisone: Short-Term Use ✓Prednisone is a corticosteroid medication that reduces inflammation and modulates immune responses. ✓It is prescribed for various inflammatory and autoimmune conditions, including but not limited to rheumatoid arthritis, lupus, asthma, and severe allergies (Lilley, 2020). ✓In a short-term therapeutic context, prednisone is often prescribed at higher dosages for a brief period (days to weeks) to control acute symptoms ✓Prednisone can be administered orally or via intravenous injection. ✓The route of administration is usually determined by the severity of the condition being treated. ✓Common side effects of short-term prednisone use may include increased appetite, weight gain, trouble sleeping, mood changes, and skin changes such as acne (Lilley, 2020). More severe side effects such as hyperglycemia, increased susceptibility to infections, osteoporosis, and ocular disorders are generally associated with long-term use. However, they should still be considered ✓during short-term therapy (Lilley, 2020). ✓It's critical to adhere to the prescribed dosage and duration and not abruptly stop the medication due to the risk of withdrawal symptoms. ✓Prednisone can interact with various other medications. ✓People with pre-existing conditions such as diabetes, hypertension, or certain infectious diseases should exercise caution when using prednisone, as it may exacerbate these conditions (Lilley, 2020). ✓Pregnant and breastfeeding women should use prednisone cautiously, as it can

The nurse is caring for a client on bed rest for a week following a right hip fracture. Which of the following findings, if noted in the client, would indicate signs of complications due to immobility? Select all that apply. An area of the client's sacrum is unable to be blanched The skin and the sclerae are yellow Crackles in the bases of the client's lungs Swelling and tenderness in the left calf The client is using the bedpan to void

hoices A, C, and D are correct. A patient on prolonged bedrest will experience complications such as decubitus ulcers (bedsores), atelectasis, and deep vein thrombosis unless preventive measures are deployed. The nurse should be aware of the signs suggestive of these complications. (Choice A) A localized area of redness (usually over a bony prominence) that does not blanch (does not turn white when pressed with a finger) and with intact skin suggests a Stage I decubitus ulcer/ pressure sore. Sacral decubitus or a pressure ulcer is a common complication with immobility. (Choice C) Crackles in an immobilized patient suggest atelectasis. The presence of crackles in bilateral lung bases indicates bibasilar atelectasis. Atelectasis refers to the collapse of the lung's alveoli (tiny air sacs). In a patient confined to prolonged bed rest, the mucus pools in the lower portions of the airway. Additionally, the diameter of bronchioles decreases in the supine position. These two factors, pooled mucus, and reduced airway size, make it difficult to clear the secretions. The supine position also places weight over the ribcage and makes the breathing labored. Therefore, the patient takes fewer breaths and is not deep enough. All these factors result in a collapse of the small airways and alveoli, leading to atelectasis. (Choice D) Swelling and tenderness in the calf suggest deep vein thrombosis (DVT). In an immobilized patient, venous stasis occurs, predisposing to venous thrombosis. DVT and pulmonary embolism are some of the severe complications of immobility. Choice B is incorrect. Scleral icterus and yellow skin suggest jaundice. Hepatitis from various causes (medications, viral infections, toxins) may result in jaundice. Liver function tests are not affected by immobility. Choice E is incorrect. Urinary incontinence is a complication of immobility. However, a patient using a bedpan to void is the desired goal and is not a complication of immobility.

37-week neonate born at 1105. Weight at birth was 7 pounds 3 ounces (3316 grams). Head circumference 33 cm; chest circumference 31 cm. The umbilical cord was clamped and cut, revealing two arteries and one vein. Vigorous cry and movement in all extremities were noted. Gluteal folds were symmetrical. Breath sounds clear, and no grunting; pink body and cyanotic extremities. Apical pulse was 119/minute; axillary temperature 96.5°F (35.8°C); respiratory rate was 50/minute. Pulse oximetry 95% on room air. APGAR at one minute was 9. The neonate is at greatest risk for______due to the neonates______ and___?

hypoglycemia....cyanosis.......axillary temp The neonate is at the greatest risk for hypoglycemia, as evidenced by cyanosis and low axillary temperature. When the neonate is cold and demonstrating cyanosis, they use more energy which burns glucose. The nurse must consider performing a heel stick for a glucose level and closely monitoring the neonate for hypoglycemia manifestations, including jittery, weak, cry, hypotonia, tachypnea, grunting, dyspnea, and tachycardia. Respiratory distress is excluded because the respiratory rate is within normal 30-60/minute parameters, pulse oximetry is optimal, and the lung sounds are clear. The neonate was born at 37 gestational weeks, which is an early term, and has an adequate weight. The head circumference is within normal limits and excludes micro- and macrocephaly. The average head circumference is 33 to 35 cm (13.2 to 14 inches), and the normal chest circumference for a term newborn is 30-36 cm (12-14 in). The gluteal folds are symmetrical, which excludes developmental dysplasia of the hip. In DDH, they would be asymmetrical. The APGAR score, done one minute and five minutes after delivery, is within the normal range. A score of 7-10 is reassuring. A vigorous cry is desired. Any cry that is high or low-pitched requires follow-up.


संबंधित स्टडी सेट्स

HIT 1.2 #A (Match the following systems of the body with their functions)

View Set

GCP for Clinical Trials Involving Drugs and Devices

View Set

Holistic Genes and Development Part 3

View Set

▶Chapter 1: Quiz#1 《Lesson 1-Lesson 3 pp. 1-30》Lesson 2

View Set

Chapter 28: Head and Spine Injuries

View Set